Finding the Value of arctan(1) Without a Calculator

  • Thread starter Thread starter kasse
  • Start date Start date
Click For Summary
The discussion focuses on finding the value of arctan(1) without a calculator, establishing that arctan(1) represents the angle whose tangent is 1. It is concluded that arctan(1) equals π/4, as tan(π/4) equals 1. The conversation emphasizes using the unit circle to identify the angle corresponding to a given tangent value. Additionally, it highlights that in a right triangle where tan(θ) equals 1, the two legs are equal, indicating that the angles are both 45 degrees. Understanding these relationships simplifies the process of determining arctan values.
kasse
Messages
383
Reaction score
1
All I know about the arctan function is that its domain of def. is the whole real line and that the range of values is (-0,5*pi , 0,5*pi), and also that arctan(0)=0.

But is there a way to know that arctan(1)=(1/4)*pi without recognizing the decimal number the calculator gives?
 
Physics news on Phys.org
I think I figured out why after some brainwork:

1=tan (arctan (1))

so because tan(pi/4)=1, arctan(1)=pi/4
 
Last edited:
i don't know if this will help but take the unit circle and reverse it. instead of looking for the angle then finding the value. look for the value then look for the angle that gives you the value. for arcsin(1), you would go along looking a the value of tan(angle) then when you find the value, the x in arcsin(x), you look at the angle that matches up with it.
 
That's easy. Just denote \arctan 1 =x and apply the \tan on both members of the equation. You'll find the eqn \tan x =1 which can easily be solved.

Daniel.
 
The simplest definition of tan(\theta) is "opposite side divided by near side" in a right triangle. If tan(\theta)= 1 then the two legs of the right triangle are the same length- it is an isosceles right triangle. What does that tell you about the angles?
 
Question: A clock's minute hand has length 4 and its hour hand has length 3. What is the distance between the tips at the moment when it is increasing most rapidly?(Putnam Exam Question) Answer: Making assumption that both the hands moves at constant angular velocities, the answer is ## \sqrt{7} .## But don't you think this assumption is somewhat doubtful and wrong?

Similar threads

  • · Replies 1 ·
Replies
1
Views
2K
  • · Replies 3 ·
Replies
3
Views
2K
  • · Replies 22 ·
Replies
22
Views
966
Replies
2
Views
1K
  • · Replies 2 ·
Replies
2
Views
3K
  • · Replies 1 ·
Replies
1
Views
3K
  • · Replies 17 ·
Replies
17
Views
4K
Replies
1
Views
2K
Replies
10
Views
2K
  • · Replies 2 ·
Replies
2
Views
2K